LSAT and Law School Admissions Forum

Get expert LSAT preparation and law school admissions advice from PowerScore Test Preparation.

User avatar
 Becca1924
  • Posts: 11
  • Joined: Dec 06, 2021
|
#93037
Hello! I am confused why A is not the correct answer? I'm not sure I fully understood it but chose it anyway. If someone could please explain why it is wrong without reference to C or E, I would really appreciate it.
 Robert Carroll
PowerScore Staff
  • PowerScore Staff
  • Posts: 1787
  • Joined: Dec 06, 2013
|
#93056
Becca1924,

The response from LSAC earlier in the thread answers your question - answer choice (A) does nothing to explain why there would be a dramatic drop in automobile thefts where only a small percentage of owners are using the device. Sure, those people trying to steal those cars are less cautious...but those cars are a small percentage, so that can't explain the big dip in thefts.

Robert Carroll
 supjeremyklein
  • Posts: 11
  • Joined: Feb 14, 2020
|
#94935
stim:
devices are undetectable and do not directly deter theft
devices are more likely to apprehend even the best car thieves
only a few people have the device in city-x yet car thefts are way down

objective:
i need to explain why car thefts are way down when only a few cars have the device

(c) is vague...
before the device was developed / after the device was developed - this doesn't tell me what happens when the devices were installed.
thieves were rarely caught / thieves were caught more than rarely - it doesn't feel strong enough to apply to the stimulus. what is this telling me about the dramatic dip in car thefts?

(e) tells me that most car thefts are done by a small group of super experienced thieves. so if most car thefts are done by super experienced, and the device makes it more likely that those super experienced will be caught. that helps explain why car thefts dipped - those super experienced either were unsuccessful or caught before they could steal more
 Adam Tyson
PowerScore Staff
  • PowerScore Staff
  • Posts: 5158
  • Joined: Apr 14, 2011
|
#95004
BOOM! Nice analysis, supjeremyklein! Put another way, we want to know how so few devices can be having such a disproportionately large impact. If they are helping us catch the people who steal most of the cars, then catching just one of them would have a large impact.

BOOM!
User avatar
 shakinotstirred
  • Posts: 3
  • Joined: Mar 25, 2024
|
#105760
So I got this question wrong by choosing choice D. A, B, C were cross-outs for me.
I went with D as I had thought that it would resolve the paradox by addressing the last sentence where it is written "... in cities where only a small percentage of car owners have the device installed, auto thefts have dropped dramatically." I thought that if a large proportion of stolen cars were stolen from people who do not "live" in the cities where they are stolen, then maybe those people (not living in the cities where the robbery was taking place) did have the devices installed in their cars, and they were just parking their car everyday (for work, or something in the like, inside the city)! (Unlike the small percentage of car owners who are living in with the device, maybe a lot of people outside the city had the devices installed).

Now I'm seeing a couple problems with choice D:

After re-reading the stimulus, my theory would fall apart as it says " The device is not yet used by a large percentage of car owners,..... " so most of my hypothetical car owners commuting into the city where the robberies decreased didn't even have the device installed anyway.

Also, the thefts would still happen! There would just be more apprehensions. This is not our case.

Also choice D doesn't resolve the paradox. The paradox in the stimulus is: how is an undetectable antitheft device (only installed by a small number of car owners) able to yield such a drastic reduction in auto thefts? Looking for a choice that will strengthen this cause / effect drawn up in the stimulus.
Now that it's clear that 1) few people have the device installed in their car (inside and outside the city) and 2) its yielding a dramatic decrease in robberies-- that even cars w/ out the device installed are also likely not being stolen (were not directly told this from the stimulus but I would say this is the case, because if it were only cars that had the device it would be a "few" cars and not a "dramatic" drop in theft).

Looking at choice E, "In most cities the MAJORITY of car thefts are committed by a FEW VERY experienced car thieves." Notice the strong language of this choice and if this can resolve our paradox.

So, if most of the car thefts in most cities are committed by the same couple (very few) experienced thieves this would resolve the paradox in one of two ways (I can think of at least): either these few car robbers likely know whats going on with this antitheft trend and have moved on from stealing cars to spending their time differently, OR even if a few of the few of them were apprehended (as the stimulus provides that .."its use greatly increases the odds of apprehending even the most experienced car thieves." ) Then even a few of the FEW experienced thieves getting caught would --> a lot less car thefts!
User avatar
 EmilyOwens
PowerScore Staff
  • PowerScore Staff
  • Posts: 27
  • Joined: Feb 27, 2024
|
#105774
shakinotstirred wrote: Mon Mar 25, 2024 6:57 pm So I got this question wrong by choosing choice D. A, B, C were cross-outs for me.
I went with D as I had thought that it would resolve the paradox by addressing the last sentence where it is written "... in cities where only a small percentage of car owners have the device installed, auto thefts have dropped dramatically." I thought that if a large proportion of stolen cars were stolen from people who do not "live" in the cities where they are stolen, then maybe those people (not living in the cities where the robbery was taking place) did have the devices installed in their cars, and they were just parking their car everyday (for work, or something in the like, inside the city)! (Unlike the small percentage of car owners who are living in with the device, maybe a lot of people outside the city had the devices installed).

Now I'm seeing a couple problems with choice D:

After re-reading the stimulus, my theory would fall apart as it says " The device is not yet used by a large percentage of car owners,..... " so most of my hypothetical car owners commuting into the city where the robberies decreased didn't even have the device installed anyway.

Also, the thefts would still happen! There would just be more apprehensions. This is not our case.

Also choice D doesn't resolve the paradox. The paradox in the stimulus is: how is an undetectable antitheft device (only installed by a small number of car owners) able to yield such a drastic reduction in auto thefts? Looking for a choice that will strengthen this cause / effect drawn up in the stimulus.
Now that it's clear that 1) few people have the device installed in their car (inside and outside the city) and 2) its yielding a dramatic decrease in robberies-- that even cars w/ out the device installed are also likely not being stolen (were not directly told this from the stimulus but I would say this is the case, because if it were only cars that had the device it would be a "few" cars and not a "dramatic" drop in theft).

Looking at choice E, "In most cities the MAJORITY of car thefts are committed by a FEW VERY experienced car thieves." Notice the strong language of this choice and if this can resolve our paradox.

So, if most of the car thefts in most cities are committed by the same couple (very few) experienced thieves this would resolve the paradox in one of two ways (I can think of at least): either these few car robbers likely know whats going on with this antitheft trend and have moved on from stealing cars to spending their time differently, OR even if a few of the few of them were apprehended (as the stimulus provides that .."its use greatly increases the odds of apprehending even the most experienced car thieves." ) Then even a few of the FEW experienced thieves getting caught would --> a lot less car thefts!
Wow! Way to set em up AND knock em down, shakin! Great work on that analysis. :)

Get the most out of your LSAT Prep Plus subscription.

Analyze and track your performance with our Testing and Analytics Package.